Difference between revisions of "Aufgaben:Exercise 3.10: Mutual Information at the BSC"

From LNTwww
(Die Seite wurde neu angelegt: „ {{quiz-Header|Buchseite=Informationstheorie/Anwendung auf die Digitalsignalübertragung }} [[File:|right|]] ===Fragebogen=== <quiz display=simple> {Multi…“)
 
 
(40 intermediate revisions by 5 users not shown)
Line 1: Line 1:
 +
{{quiz-Header|Buchseite=Information_Theory/Application_to_Digital_Signal_Transmission
 +
}}
 +
 +
[[File:P_ID2787__Inf_A_3_9.png|right|frame|BSC model considered]]
 +
We consider the&nbsp; [[Channel_Coding/Kanalmodelle_und_Entscheiderstrukturen#Binary_Symmetric_Channel_.E2.80.93_BSC|Binary Symmetric Channel]]&nbsp; $\rm (BSC)$. The parameter values are valid for the whole exercise:
 +
* Crossover probability: &nbsp; $\varepsilon = 0.1$,
 +
* Probability for&nbsp; $0$: &nbsp;  $p_0 = 0.2$,
 +
* Probability for&nbsp; $1$: &nbsp;  $p_1 = 0.8$.
 +
 +
 +
Thus the probability mass function of the source is: &nbsp; $P_X(X)= (0.2 , \ 0.8)$&nbsp; and for the source entropy applies:
 +
:$$H(X) = p_0 \cdot {\rm log}_2 \hspace{0.1cm} \frac{1}{p_0} + p_1\cdot {\rm log}_2 \hspace{0.1cm} \frac{1}{p_1} = H_{\rm bin}(0.2)={ 0.7219\,{\rm bit}} \hspace{0.05cm}.$$
 +
 +
The task is to determine:
 +
* the probability function of the sink:
 +
:$$P_Y(Y) = (\hspace{0.05cm}P_Y(0)\hspace{0.05cm}, \ \hspace{0.05cm} P_Y(1)\hspace{0.05cm})
 +
\hspace{0.05cm},$$
 +
* the joint probability function:
 +
:$$P_{XY}(X, Y) = \begin{pmatrix}
 +
p_{00}  & p_{01}\\
 +
p_{10}  & p_{11}
 +
\end{pmatrix}  \hspace{0.05cm},$$
 +
* the mutual information:
 +
:$$I(X;Y) = {\rm E} \hspace{-0.08cm}\left [ \hspace{0.02cm}{\rm log}_2 \hspace{0.1cm} \frac{P_{XY}(X, Y)}
 +
{P_{X}(X) \cdot P_{Y}(Y) }\right ]  \hspace{0.05cm},$$
 +
*the equivocation:
 +
:$$H(X \hspace{-0.1cm}\mid \hspace{-0.1cm} Y) =  {\rm E} \hspace{0.02cm} \big [ \hspace{0.02cm} {\rm log}_2 \hspace{0.1cm} \frac{1}{P_{\hspace{0.03cm}X \mid \hspace{0.03cm} Y} (X \hspace{-0.05cm}\mid \hspace{-0.05cm} Y)} \big ] \hspace{0.05cm},$$
 +
*the irrelevance:
 +
:$$H(Y \hspace{-0.1cm}\mid \hspace{-0.1cm} X) =  {\rm E} \hspace{0.02cm} \big [ \hspace{0.02cm} {\rm log}_2 \hspace{0.1cm} \frac{1}{P_{\hspace{0.03cm}Y \mid \hspace{0.03cm} X} (Y \hspace{-0.05cm}\mid \hspace{-0.05cm} X)} \big ] \hspace{0.05cm}.$$
 +
 +
 +
 +
 +
  
{{quiz-Header|Buchseite=Informationstheorie/Anwendung auf die Digitalsignalübertragung
 
}}
 
  
[[File:|right|]]
+
Hints:
 +
*The exercise belongs to the chapter&nbsp; [[Information_Theory/Anwendung_auf_die_Digitalsignalübertragung|Application to Digital Signal Transmission]].
 +
*Reference is made in particular to the page&nbsp;    [[Information_Theory/Anwendung_auf_die_Digitalsignalübertragung#Calculation_of_the_mutual_information_for_the_binary_channel|Mutual information calculation for the binary channel]].
 +
*In&nbsp; [[Aufgaben:Aufgabe_3.10Z:_BSC–Kanalkapazität|Exercise 3.10Z]]&nbsp; the channel capacity&nbsp; $C_{\rm BSC }$&nbsp; of the BSC model is calculated.
 +
*This results as the maximum mutual information&nbsp; $I(X;\ Y)$&nbsp;  by maximization with respect to the probabilities&nbsp; $p_0$&nbsp; or&nbsp; $p_1 = 1 - p_0$.
 +
  
  
===Fragebogen===
+
===Questions===
  
 
<quiz display=simple>
 
<quiz display=simple>
{Multiple-Choice Frage
 
|type="[]"}
 
- Falsch
 
+ Richtig
 
  
 +
{Calculate the joint probabilities&nbsp; $P_{ XY }(X, Y)$
 +
|type="{}"}
 +
$P_{ XY }(0, 0) \ = \ $ { 0.18 3% }
 +
$P_{ XY }(0, 1) \ = \ $ { 0.02 3% }
 +
$P_{ XY }(1, 0) \ = \ $ { 0.08 3% }
 +
$P_{ XY }(1, 1) \ = \ $ { 0.72 3% }
 +
 +
{What is the probability mass function&nbsp; $P_Y(Y)$&nbsp; of the sink?
 +
|type="{}"}
 +
$P_Y(0)\ = \ $ { 0.26 3% }
 +
$P_Y(1) \ = \ $ { 0.74 3% }
 +
 +
{What is the value of the mutual information&nbsp; $I(X;\
 +
Y)$?
 +
|type="{}"}
 +
$I(X; Y)\ = \ $ { 0.3578 3% } $\ \rm bit$
  
{Input-Box Frage
+
{Which value results for the equivocation&nbsp; $H(X|Y)$?
 
|type="{}"}
 
|type="{}"}
$\alpha$ = { 0.3 }
+
$H(X|Y) \ = \ $ { 0.3642 3% } $\ \rm bit$
 +
 
 +
 
 +
{Which statement is true for the sink entropy&nbsp; $H(Y)$&nbsp;?
 +
|type="[]"}
 +
- $H(Y)$&nbsp; is never greater than&nbsp; $H(X)$.
 +
+ $H(Y)$&nbsp; is never smaller than&nbsp; $H(X)$.
  
 +
{Which statement is true for the irrelevance&nbsp; $H(Y|X)$&nbsp;?
 +
|type="[]"}
 +
- $H(Y|X)$&nbsp; is never larger than the equivocation&nbsp; $H(X|Y)$.
 +
+ $H(Y|X)$&nbsp; is never smaller than the equivocation&nbsp; $H(X|Y)$.
  
  
 
</quiz>
 
</quiz>
  
===Musterlösung===
+
===Solution===
 
{{ML-Kopf}}
 
{{ML-Kopf}}
'''1.'''
+
'''(1)'''&nbsp; The following applies in general or with the numerical values&nbsp; $p_0 = 0.2$&nbsp; and&nbsp; $\varepsilon = 0.1$ for the quantities sought:
'''2.'''
+
:$$P_{XY}(0, 0) = p_0 \cdot (1 - \varepsilon)
'''3.'''
+
\hspace{0.15cm} \underline {=0.18} \hspace{0.05cm}, \hspace{0.5cm}
'''4.'''
+
P_{XY}(0, 1) = p_0 \cdot \varepsilon
'''5.'''
+
\hspace{0.15cm} \underline {=0.02} \hspace{0.05cm},$$
'''6.'''
+
:$$P_{XY}(1, 0) = p_1 \cdot \varepsilon
'''7.'''
+
\hspace{0.15cm} \underline {=0.08} \hspace{0.05cm}, \hspace{1.55cm}
 +
P_{XY}(1, 1) = p_1 \cdot (1 - \varepsilon)
 +
\hspace{0.15cm} \underline {=0.72} \hspace{0.05cm}.$$
 +
 
 +
 
 +
 
 +
'''(2)'''&nbsp; In general:
 +
:$$P_Y(Y) = \big [ {\rm Pr}( Y = 0)\hspace{0.05cm}, {\rm Pr}( Y = 1) \big ] = \big ( p_0\hspace{0.05cm}, p_1 \big ) \cdot \begin{pmatrix} 1 - \varepsilon & \varepsilon\\ \varepsilon & 1 - \varepsilon \end{pmatrix}.$$
 +
This gives the following numerical values:
 +
:$$ {\rm Pr}( Y = 0)= p_0 \cdot (1 - \varepsilon) + p_1 \cdot \varepsilon = 0.2 \cdot 0.9 + 0.8 \cdot 0.1 \hspace{0.15cm} \underline {=0.26} \hspace{0.05cm},$$
 +
:$${\rm Pr}( Y = 1)= p_0 \cdot \varepsilon + p_1 \cdot (1 - \varepsilon) = 0.2 \cdot 0.1 + 0.8 \cdot 0.9 \hspace{0.15cm} \underline {=0.74} \hspace{0.05cm}.$$
 +
 
 +
 
 +
 
 +
'''(3)'''&nbsp; For the mutual information, according to the definition with&nbsp; $p_0 = 0.2$,&nbsp; $p_1 = 0.8$&nbsp; and&nbsp; $\varepsilon = 0.1$:
 +
:$$I(X;Y) = {\rm E} \hspace{-0.08cm}\left [ \hspace{0.02cm}{\rm log}_2 \hspace{0.08cm} \frac{P_{XY}(X, Y)} {P_{X}(X) \hspace{-0.05cm}\cdot \hspace{-0.05cm} P_{Y}(Y) }\right ] \hspace{0.3cm} \Rightarrow$$
 +
:$$I(X;Y)  = 0.18 \cdot {\rm log}_2 \hspace{0.1cm} \frac{0.18}{0.2 \hspace{-0.05cm}\cdot \hspace{-0.05cm} 0.26} + 0.02 \cdot {\rm log}_2 \hspace{0.08cm} \frac{0.02}{0.2 \hspace{-0.05cm}\cdot \hspace{-0.05cm} 0.74}
 +
+ 0.08 \cdot {\rm log}_2 \hspace{0.08cm} \frac{0.08}{0.8 \hspace{-0.05cm}\cdot \hspace{-0.05cm} 0.26} + 0.72 \cdot {\rm log}_2 \hspace{0.08cm} \frac{0.72}{0.8 \hspace{-0.05cm}\cdot \hspace{-0.05cm} 0.74} \hspace{0.15cm} \underline {=0.3578\,{\rm bit}} \hspace{0.05cm}.$$
 +
 
 +
 
 +
 
 +
'''(4)'''&nbsp; With the source entropy&nbsp; $H(X)$&nbsp; given, we obtain for the equivocation:
 +
:$$H(X \hspace{-0.1cm}\mid \hspace{-0.1cm} Y) = H(X) - I(X;Y) = 0.7219 - 0.3578 \hspace{0.15cm} \underline {=0.3642\,{\rm bit}} \hspace{0.05cm}.$$
 +
*However, one could also apply the general definition with the inference probabilities&nbsp; $P_{X|Y}(⋅)$&nbsp;:
 +
:$$H(X \hspace{-0.1cm}\mid \hspace{-0.1cm} Y) = {\rm E} \hspace{0.02cm} \left [ \hspace{0.05cm} {\rm log}_2 \hspace{0.1cm} \frac{1}{P_{\hspace{0.03cm}X \mid \hspace{0.03cm} Y} (X \hspace{-0.05cm}\mid \hspace{-0.05cm} Y)} \hspace{0.05cm}\right ] = {\rm E} \hspace{0.02cm} \left [ \hspace{0.05cm} {\rm log}_2 \hspace{0.1cm} \frac{P_Y(Y)}{P_{XY} (X, Y)} \hspace{0.05cm} \right ] \hspace{0.05cm}$$
 +
 
 +
*In the example, the same result&nbsp; $H(X|Y) = 0.3642 \ \rm bit$&nbsp; is also obtained according to this calculation rule:
 +
:$$H(X \hspace{-0.1cm}\mid \hspace{-0.1cm} Y) = 0.18 \cdot {\rm log}_2 \hspace{0.1cm} \frac{0.26}{0.18} + 0.02 \cdot {\rm log}_2 \hspace{0.1cm} \frac{0.74}{0.02} + 0.08 \cdot {\rm log}_2 \hspace{0.1cm} \frac{0.26}{0.08} + 0.72 \cdot {\rm log}_2 \hspace{0.1cm} \frac{0.74}{0.72} \hspace{0.05cm}.$$
 +
 
 +
 
 +
 
 +
'''(5)'''&nbsp; Correct is the <u>proposed solution 2:</u>
 +
*In the case of disturbed transmission&nbsp; $(ε > 0)$&nbsp; the uncertainty regarding the sink is always greater than the uncertainty regarding the source.&nbsp; One obtains here as a numerical value:
 +
:$$H(Y) = H_{\rm bin}(0.26)={ 0.8268\,{\rm bit}} \hspace{0.05cm}.$$
 +
*With error-free transmission&nbsp; $(ε = 0)$,&nbsp; on the other hand,&nbsp; $P_Y(⋅) = P_X(⋅)$&nbsp; and&nbsp; $H(Y) = H(X)$&nbsp; would apply.
 +
 
 +
 
 +
 
 +
'''(6)'''&nbsp; Here, too, the <u>second proposed solution</u> is correct:
 +
*Because of&nbsp; $I(X;Y) = H(X) - H(X \hspace{-0.1cm}\mid \hspace{-0.1cm} Y) = H(Y) - H(Y \hspace{-0.1cm}\mid \hspace{-0.1cm} X)$&nbsp;,&nbsp; $H(Y|X)$&nbsp; is greater than&nbsp; $H(X|Y)$ by the same magnitude that&nbsp; $H(Y)$&nbsp; is greater than&nbsp; $H(X)$:
 +
:$$H(Y \hspace{-0.1cm}\mid \hspace{-0.1cm} X) = H(Y) -I(X;Y) = 0.8268 - 0.3578 ={ 0.469\,{\rm bit}} \hspace{0.05cm}$$
 +
*Direct calculation gives the same result&nbsp; $H(Y|X) = 0.469\ \rm  bit$:
 +
:$$H(Y \hspace{-0.1cm}\mid \hspace{-0.1cm} X) = {\rm E} \hspace{0.02cm} \left [ \hspace{0.02cm} {\rm log}_2 \hspace{0.1cm} \frac{1}{P_{\hspace{0.03cm}Y \mid \hspace{0.03cm} X} (Y \hspace{-0.05cm}\mid \hspace{-0.05cm} X)} \right ] = 0.18 \cdot {\rm log}_2 \hspace{0.1cm} \frac{1}{0.9} + 0.02 \cdot {\rm log}_2 \hspace{0.1cm} \frac{1}{0.1} + 0.08 \cdot {\rm log}_2 \hspace{0.1cm} \frac{1}{0.1} + 0.72 \cdot {\rm log}_2 \hspace{0.1cm} \frac{1}{0.9} \hspace{0.05cm}.$$
 +
 
 
{{ML-Fuß}}
 
{{ML-Fuß}}
  
  
  
[[Category:Aufgaben zu Informationstheorie|^3.3 Anwendung auf die Digitalsignalübertragung^]]
+
[[Category:Information Theory: Exercises|^3.3 Application to Digital Signal Transmission^]]

Latest revision as of 07:05, 18 September 2022

BSC model considered

We consider the  Binary Symmetric Channel  $\rm (BSC)$. The parameter values are valid for the whole exercise:

  • Crossover probability:   $\varepsilon = 0.1$,
  • Probability for  $0$:   $p_0 = 0.2$,
  • Probability for  $1$:   $p_1 = 0.8$.


Thus the probability mass function of the source is:   $P_X(X)= (0.2 , \ 0.8)$  and for the source entropy applies:

$$H(X) = p_0 \cdot {\rm log}_2 \hspace{0.1cm} \frac{1}{p_0} + p_1\cdot {\rm log}_2 \hspace{0.1cm} \frac{1}{p_1} = H_{\rm bin}(0.2)={ 0.7219\,{\rm bit}} \hspace{0.05cm}.$$

The task is to determine:

  • the probability function of the sink:
$$P_Y(Y) = (\hspace{0.05cm}P_Y(0)\hspace{0.05cm}, \ \hspace{0.05cm} P_Y(1)\hspace{0.05cm}) \hspace{0.05cm},$$
  • the joint probability function:
$$P_{XY}(X, Y) = \begin{pmatrix} p_{00} & p_{01}\\ p_{10} & p_{11} \end{pmatrix} \hspace{0.05cm},$$
  • the mutual information:
$$I(X;Y) = {\rm E} \hspace{-0.08cm}\left [ \hspace{0.02cm}{\rm log}_2 \hspace{0.1cm} \frac{P_{XY}(X, Y)} {P_{X}(X) \cdot P_{Y}(Y) }\right ] \hspace{0.05cm},$$
  • the equivocation:
$$H(X \hspace{-0.1cm}\mid \hspace{-0.1cm} Y) = {\rm E} \hspace{0.02cm} \big [ \hspace{0.02cm} {\rm log}_2 \hspace{0.1cm} \frac{1}{P_{\hspace{0.03cm}X \mid \hspace{0.03cm} Y} (X \hspace{-0.05cm}\mid \hspace{-0.05cm} Y)} \big ] \hspace{0.05cm},$$
  • the irrelevance:
$$H(Y \hspace{-0.1cm}\mid \hspace{-0.1cm} X) = {\rm E} \hspace{0.02cm} \big [ \hspace{0.02cm} {\rm log}_2 \hspace{0.1cm} \frac{1}{P_{\hspace{0.03cm}Y \mid \hspace{0.03cm} X} (Y \hspace{-0.05cm}\mid \hspace{-0.05cm} X)} \big ] \hspace{0.05cm}.$$




Hints:


Questions

1

Calculate the joint probabilities  $P_{ XY }(X, Y)$

$P_{ XY }(0, 0) \ = \ $

$P_{ XY }(0, 1) \ = \ $

$P_{ XY }(1, 0) \ = \ $

$P_{ XY }(1, 1) \ = \ $

2

What is the probability mass function  $P_Y(Y)$  of the sink?

$P_Y(0)\ = \ $

$P_Y(1) \ = \ $

3

What is the value of the mutual information  $I(X;\ Y)$?

$I(X; Y)\ = \ $

$\ \rm bit$

4

Which value results for the equivocation  $H(X|Y)$?

$H(X|Y) \ = \ $

$\ \rm bit$

5

Which statement is true for the sink entropy  $H(Y)$ ?

$H(Y)$  is never greater than  $H(X)$.
$H(Y)$  is never smaller than  $H(X)$.

6

Which statement is true for the irrelevance  $H(Y|X)$ ?

$H(Y|X)$  is never larger than the equivocation  $H(X|Y)$.
$H(Y|X)$  is never smaller than the equivocation  $H(X|Y)$.


Solution

(1)  The following applies in general or with the numerical values  $p_0 = 0.2$  and  $\varepsilon = 0.1$ for the quantities sought:

$$P_{XY}(0, 0) = p_0 \cdot (1 - \varepsilon) \hspace{0.15cm} \underline {=0.18} \hspace{0.05cm}, \hspace{0.5cm} P_{XY}(0, 1) = p_0 \cdot \varepsilon \hspace{0.15cm} \underline {=0.02} \hspace{0.05cm},$$
$$P_{XY}(1, 0) = p_1 \cdot \varepsilon \hspace{0.15cm} \underline {=0.08} \hspace{0.05cm}, \hspace{1.55cm} P_{XY}(1, 1) = p_1 \cdot (1 - \varepsilon) \hspace{0.15cm} \underline {=0.72} \hspace{0.05cm}.$$


(2)  In general:

$$P_Y(Y) = \big [ {\rm Pr}( Y = 0)\hspace{0.05cm}, {\rm Pr}( Y = 1) \big ] = \big ( p_0\hspace{0.05cm}, p_1 \big ) \cdot \begin{pmatrix} 1 - \varepsilon & \varepsilon\\ \varepsilon & 1 - \varepsilon \end{pmatrix}.$$

This gives the following numerical values:

$$ {\rm Pr}( Y = 0)= p_0 \cdot (1 - \varepsilon) + p_1 \cdot \varepsilon = 0.2 \cdot 0.9 + 0.8 \cdot 0.1 \hspace{0.15cm} \underline {=0.26} \hspace{0.05cm},$$
$${\rm Pr}( Y = 1)= p_0 \cdot \varepsilon + p_1 \cdot (1 - \varepsilon) = 0.2 \cdot 0.1 + 0.8 \cdot 0.9 \hspace{0.15cm} \underline {=0.74} \hspace{0.05cm}.$$


(3)  For the mutual information, according to the definition with  $p_0 = 0.2$,  $p_1 = 0.8$  and  $\varepsilon = 0.1$:

$$I(X;Y) = {\rm E} \hspace{-0.08cm}\left [ \hspace{0.02cm}{\rm log}_2 \hspace{0.08cm} \frac{P_{XY}(X, Y)} {P_{X}(X) \hspace{-0.05cm}\cdot \hspace{-0.05cm} P_{Y}(Y) }\right ] \hspace{0.3cm} \Rightarrow$$
$$I(X;Y) = 0.18 \cdot {\rm log}_2 \hspace{0.1cm} \frac{0.18}{0.2 \hspace{-0.05cm}\cdot \hspace{-0.05cm} 0.26} + 0.02 \cdot {\rm log}_2 \hspace{0.08cm} \frac{0.02}{0.2 \hspace{-0.05cm}\cdot \hspace{-0.05cm} 0.74} + 0.08 \cdot {\rm log}_2 \hspace{0.08cm} \frac{0.08}{0.8 \hspace{-0.05cm}\cdot \hspace{-0.05cm} 0.26} + 0.72 \cdot {\rm log}_2 \hspace{0.08cm} \frac{0.72}{0.8 \hspace{-0.05cm}\cdot \hspace{-0.05cm} 0.74} \hspace{0.15cm} \underline {=0.3578\,{\rm bit}} \hspace{0.05cm}.$$


(4)  With the source entropy  $H(X)$  given, we obtain for the equivocation:

$$H(X \hspace{-0.1cm}\mid \hspace{-0.1cm} Y) = H(X) - I(X;Y) = 0.7219 - 0.3578 \hspace{0.15cm} \underline {=0.3642\,{\rm bit}} \hspace{0.05cm}.$$
  • However, one could also apply the general definition with the inference probabilities  $P_{X|Y}(⋅)$ :
$$H(X \hspace{-0.1cm}\mid \hspace{-0.1cm} Y) = {\rm E} \hspace{0.02cm} \left [ \hspace{0.05cm} {\rm log}_2 \hspace{0.1cm} \frac{1}{P_{\hspace{0.03cm}X \mid \hspace{0.03cm} Y} (X \hspace{-0.05cm}\mid \hspace{-0.05cm} Y)} \hspace{0.05cm}\right ] = {\rm E} \hspace{0.02cm} \left [ \hspace{0.05cm} {\rm log}_2 \hspace{0.1cm} \frac{P_Y(Y)}{P_{XY} (X, Y)} \hspace{0.05cm} \right ] \hspace{0.05cm}$$
  • In the example, the same result  $H(X|Y) = 0.3642 \ \rm bit$  is also obtained according to this calculation rule:
$$H(X \hspace{-0.1cm}\mid \hspace{-0.1cm} Y) = 0.18 \cdot {\rm log}_2 \hspace{0.1cm} \frac{0.26}{0.18} + 0.02 \cdot {\rm log}_2 \hspace{0.1cm} \frac{0.74}{0.02} + 0.08 \cdot {\rm log}_2 \hspace{0.1cm} \frac{0.26}{0.08} + 0.72 \cdot {\rm log}_2 \hspace{0.1cm} \frac{0.74}{0.72} \hspace{0.05cm}.$$


(5)  Correct is the proposed solution 2:

  • In the case of disturbed transmission  $(ε > 0)$  the uncertainty regarding the sink is always greater than the uncertainty regarding the source.  One obtains here as a numerical value:
$$H(Y) = H_{\rm bin}(0.26)={ 0.8268\,{\rm bit}} \hspace{0.05cm}.$$
  • With error-free transmission  $(ε = 0)$,  on the other hand,  $P_Y(⋅) = P_X(⋅)$  and  $H(Y) = H(X)$  would apply.


(6)  Here, too, the second proposed solution is correct:

  • Because of  $I(X;Y) = H(X) - H(X \hspace{-0.1cm}\mid \hspace{-0.1cm} Y) = H(Y) - H(Y \hspace{-0.1cm}\mid \hspace{-0.1cm} X)$ ,  $H(Y|X)$  is greater than  $H(X|Y)$ by the same magnitude that  $H(Y)$  is greater than  $H(X)$:
$$H(Y \hspace{-0.1cm}\mid \hspace{-0.1cm} X) = H(Y) -I(X;Y) = 0.8268 - 0.3578 ={ 0.469\,{\rm bit}} \hspace{0.05cm}$$
  • Direct calculation gives the same result  $H(Y|X) = 0.469\ \rm bit$:
$$H(Y \hspace{-0.1cm}\mid \hspace{-0.1cm} X) = {\rm E} \hspace{0.02cm} \left [ \hspace{0.02cm} {\rm log}_2 \hspace{0.1cm} \frac{1}{P_{\hspace{0.03cm}Y \mid \hspace{0.03cm} X} (Y \hspace{-0.05cm}\mid \hspace{-0.05cm} X)} \right ] = 0.18 \cdot {\rm log}_2 \hspace{0.1cm} \frac{1}{0.9} + 0.02 \cdot {\rm log}_2 \hspace{0.1cm} \frac{1}{0.1} + 0.08 \cdot {\rm log}_2 \hspace{0.1cm} \frac{1}{0.1} + 0.72 \cdot {\rm log}_2 \hspace{0.1cm} \frac{1}{0.9} \hspace{0.05cm}.$$